Đến nội dung

Hình ảnh

$\boxed{Topic}$ ÔN THI VÀO THPT CHUYÊN TOÁN NĂM HỌC 2017-2018

ôn thi vào thpt chuyên 2017 ôn thi vào thpt chuyên toán

  • Please log in to reply
Chủ đề này có 136 trả lời

#101
viet9a14124869

viet9a14124869

    Trung úy

  • Thành viên
  • 903 Bài viết

$\boxed{\text{Lời giải bài 15}}$

Đặt $a+b-c-1=2x^{3}, b+c-a-1=2y^{3}, c+a-b-1=2z^{3}$. Bất đẳng thức cần chứng minh tương đương $xyz\leq 1$

Từ cách đặt trên ta có: $\frac{1}{x^{3}+y^{3}+1}+\frac{1}{z^{3}+y^{3}+1}+\frac{1}{x^{3}+z^{3}+1}=1$

Giả sử $xyz>1\Rightarrow xyz(x+y)+ z>x+y+z$ $(1)$

Mặt khác $xy(x+y)+1\leq x^{3}+y^{3}+1\Rightarrow z[xy(x+y)+1]\leq z(x^{3}+y^{3}+1)\Rightarrow xyz(x+y)+z\leq z(x^{3}+y^{3}+1)$ $(2)$

Từ $(1), (2)$ suy ra $x+y+z< z(x^{3}+y^{3}+1)\Rightarrow \frac{z}{x+y+z}>\frac{1}{x^{3}+y^{3}+1}$

Tương tự ta cũng có: $\frac{x}{x+y+z}> \frac{1}{z^{3}+y^{3}+1}$

                                   $\frac{y}{x+y+z}> \frac{1}{x^{3}+z^{3}+1}$

Cộng các bất đẳng thức trên ta có: $1=\frac{1}{x^{3}+y^{3}+1}+\frac{1}{z^{3}+y^{3}+1}+\frac{1}{x^{3}+z^{3}+1}< 1$ (Vô lí) 

Vậy điều giả sử là sai $\Rightarrow xyz\leq 1\Rightarrow ĐPCM$

                                   

Đây là đề thi tỉnh mình hôm nọ T___T 

Sao bạn biết cách đặt này vậy ,,,,, !!! 


                                                                    SÓNG BẮT ĐẦU TỪ GIÓ

                                                                    GIÓ BẮT ĐẦU TỪ ĐÂU ?

                                                                    ANH CŨNG KHÔNG BIẾT NỮA 

                                                                    KHI NÀO...? TA YÊU NHAU .


#102
NHoang1608

NHoang1608

    Sĩ quan

  • Thành viên
  • 375 Bài viết

Đây là đề thi tỉnh mình hôm nọ T___T 

Sao bạn biết cách đặt này vậy ,,,,, !!! 

Đặt kiểu này khá lạ $x^{3}$ để dùng bđt phụ cho đẹp mắt còn quan trọng là con số 2 để khử con 2 ở dưới mẫu và đảm bảo $x=1$ thay vì $x=\sqrt[3]{2}$ ( nhận thấy $a+b-c-1=2$ ). Có thể có cách khác đẹp mắt hơn để chứng minh $xyz\leq 1$ nhưng mình chưa tìm ra. Ngoài ra bạn có thể sử dụng đẳng thức $(a+b+c)(ab+bc+ca)-abc=(a+b)(b+c)(c+a)$ để chứng minh sau khi quy đồng giả thiết.

Bạn thi tỉnh nào vậy.


Bài viết đã được chỉnh sửa nội dung bởi NHoang1608: 31-03-2017 - 13:33

The greatest danger for most of us is not that our aim is too high and we miss it, but that it is too low and we reach it.

----- Michelangelo----


#103
viet9a14124869

viet9a14124869

    Trung úy

  • Thành viên
  • 903 Bài viết

Đặt kiểu này khá lạ $x^{3}$ để dùng bđt phụ cho đẹp mắt còn quan trọng là con số 2 để khử con 2 ở dưới mẫu và đảm bảo dấu bằng vì nhận thấy a=2 mà. Có thể có cách khác đẹp mắt hơn để chứng minh $xyz\leq 1$ nhưng mình chưa tìm ra. Ngoài ra bạn có thể sử dụng đẳng thức $(a+b+c)(ab+bc+ca)-abc=(a+b)(b+c)(c+a)$ để chứng minh sau khi quy đồng giả thiết.

Bạn thi tỉnh nào vậy.

Vĩnh Phúc bạn ạ ,,,, Mình bỏ câu này nên được có giải Nhì thôi ,,, bạn trình bày luôn cách đồng quy có được không,,,mình nghĩ là ngược dấu :P 


                                                                    SÓNG BẮT ĐẦU TỪ GIÓ

                                                                    GIÓ BẮT ĐẦU TỪ ĐÂU ?

                                                                    ANH CŨNG KHÔNG BIẾT NỮA 

                                                                    KHI NÀO...? TA YÊU NHAU .


#104
Minhnksc

Minhnksc

    Sĩ quan

  • Điều hành viên OLYMPIC
  • 302 Bài viết

$\boxed{18}$: cho các số thực dương a, b, c thỏa $\sqrt{b^2+c^2}+\sqrt{c^2+a^2}+\sqrt{a^2+b^2}=\sqrt{2015}$

tìm min $P=\frac{a^2}{b+c}+\frac{b^2}{a+c}+\frac{c^2}{a+b}$

câu này mình giải dài lắm nên muốn tham khảo lời giải của các bạn

Vĩnh Phúc bạn ạ ,,,, Mình bỏ câu này nên được có giải Nhì thôi ,,, bạn trình bày luôn cách đồng quy có được không,,,mình nghĩ là ngược dấu :PVi

Vĩnh Phúc đã thi rồi à, Hà Nam mình tuần nữa mới thi cơ, mà bạn up đề Vĩnh Phúc lên được không.

P/s: Hóng câu hình đề Vĩnh Phúc quá


Bài viết đã được chỉnh sửa nội dung bởi Minhnksc: 31-03-2017 - 21:14

Sống khỏe và sống tốt :D


#105
HoangKhanh2002

HoangKhanh2002

    Sĩ quan

  • Thành viên
  • 483 Bài viết

$\boxed{16}$ Cho $a, b, c$ là các số thực dương. Tìm giá trị nhỏ nhất của biểu thức

                     $P=\frac{a^{2}}{(a+b)^{2}}+\frac{b^{2}}{(b+c)^{2}}+\frac{c}{4a}$.

$\boxed{17}$ Giải phương trình $\sqrt{5-3x}+\sqrt{x+1}=\sqrt{3x^{2}-4x+4}$.

$\boxed{16}$

Ta có: $P=\frac{a^{2}}{(a+b)^{2}}+\frac{b^{2}}{(b+c)^{2}}+\frac{c}{4a}=\frac{1}{\left ( 1+\frac{b}{a} \right )^2}+\frac{1}{\left ( 1+\frac{c}{b} \right )^2}+\frac{1}{4}.\frac{b}{a}.\frac{c}{b}\geq \frac{1}{1+\frac{b}{a}.\frac{c}{b}}+\frac{1}{4}.\frac{b}{a}.\frac{c}{b}=\left ( \frac{\frac{b}{a}.\frac{c}{b}+1}{4}+\frac{1}{1+\frac{b}{a}.\frac{c}{b}} \right )-\frac{1}{4}\geq 1-\frac{1}{4}=\frac{3}{4}$

Vì có BĐT: $\frac{1}{(1+a)^2}+\frac{1}{(1+b)^2}\geq \frac{1}{1+ab}$

$\boxed{17}$

ĐK: $-1\leq x\leq \frac{5}{3}$

Phương trình đã cho tương đương với: $6-2x+2\sqrt{(5-3x)(x+1)}=3x^2-4x+4\Leftrightarrow -3x^2+2x+2\sqrt{-3x^2+2x+5}=0\Leftrightarrow (-3x^2+2x+5)+2\sqrt{-3x^2+2x+5}-3=0\Leftrightarrow \begin{bmatrix} \sqrt{-3x^2+2x+5}=-3(KTM)\\ \sqrt{-3x^2+2x+5}=1\Rightarrow x=\frac{1\pm \sqrt{13}}{3}(TM) \end{bmatrix}$

PS: Dạo này bận quá trời.... Không có thời gian...Giờ đi học cái đã....


Bài viết đã được chỉnh sửa nội dung bởi HoangKhanh2002: 31-03-2017 - 18:48


#106
NHoang1608

NHoang1608

    Sĩ quan

  • Thành viên
  • 375 Bài viết

$\boxed{16}$: cho các số thực dương a, b, c thỏa $\sqrt{b^2+c^2}+\sqrt{c^2+a^2}+\sqrt{a^2+b^2}=\sqrt{2015}$

tìm min $P=\frac{a^2}{b+c}+\frac{b^2}{a+c}+\frac{c^2}{a+b}$

câu này mình giải dài lắm nên muốn tham khảo lời giải của các bạn

Vĩnh Phúc đã thi rồi à, Hà Nam mình tuần nữa mới thi cơ, mà bạn up đề Vĩnh Phúc lên được không.

P/s: Hóng câu hình đề Vĩnh Phúc quá

$\boxed{\text{Lời giải bài 18}}$

Đặt $\sqrt{a^{2}+b^{2}}=x$,$\sqrt{b^{2}+c^{2}}$,$\sqrt{c^{2}+a^{2}}$

Suy ra $x^{2}+y^{2}-z^{2}=2b^{2}$ và $\sqrt{a^{2}+b^{2}}\geq \sqrt{\frac{(a+b)^{2}}{2}}=\frac{a+b}{\sqrt{2}} \Rightarrow \frac{\sqrt{a^{2}+b^{2}}}{\sqrt{2}} \geq a+b$ $(1)$

$P \frac{x^{2}+y^{2}-z^{2}}{2(b+c)}+\frac{x^{2}+z^{2}-y^{2}}{2(c+a)}+\frac{z^{2}+y^{2}-x^{2}}{2(a+b)}$

Từ $(1)$ suy ra $P \geq \frac{x^{2}+y^{2}-z^{2}}{2\sqrt{2}z}+\frac{x^{2}+z^{2}-y^{2}}{2\sqrt{2}y}+\frac{z^{2}+y^{2}-x^{2}}{2\sqrt{2}x}$

                                               $= \frac{1}{2\sqrt{2}} (\frac{x^{2}}{y}+\frac{z^{2}}{y}-y+\frac{y^{2}}{x}+\frac{z^{2}}{x}-x+\frac{x^{2}}{z}+\frac{y^{2}}{z}-z)$

                                               $= \frac{1}{2\sqrt{2}} (\frac{x^{2}}{y}+y+\frac{z^{2}}{y}+y-3y+\frac{y^{2}}{x}+x+\frac{z^{2}}{x}+x-3x+\frac{x^{2}}{z}+z+\frac{y^{2}}{z}+z-3z)$

Mặt khác áp dụng bđt $AM-GM$ thì ta có: $\frac{x^{2}}{y}+y \geq 2x$, $\frac{z^{2}}{y}+y \geq 2z$, tương tự.

Suy ra $ \frac{1}{2\sqrt{2}} (\frac{x^{2}}{y}+y+\frac{z^{2}}{y}+y-3y+\frac{y^{2}}{x}+x+\frac{z^{2}}{x}+x-3x+\frac{x^{2}}{z}+z+\frac{y^{2}}{z}+z-3z) \geq \frac{1}{2\sqrt{2}}(x+y+z)= \frac{1}{2\sqrt{2}}\sqrt{2015}$

Đẳng thức xảy ra khi $x=y=z$

 

 

P/s: Với lại bạn chỉnh lại số bài thành 18 đi mình ms up 2 bài ở trang 5 đó.


Bài viết đã được chỉnh sửa nội dung bởi NHoang1608: 31-03-2017 - 18:48

The greatest danger for most of us is not that our aim is too high and we miss it, but that it is too low and we reach it.

----- Michelangelo----


#107
tienduc

tienduc

    Thiếu úy

  • Điều hành viên THCS
  • 580 Bài viết

$\boxed{16}$: cho các số thực dương a, b, c thỏa $\sqrt{b^2+c^2}+\sqrt{c^2+a^2}+\sqrt{a^2+b^2}=\sqrt{2015}$

tìm min $P=\frac{a^2}{b+c}+\frac{b^2}{a+c}+\frac{c^2}{a+b}$

$\boxed{\text{Lời giải bài 18}}$

Áp dụng BĐT $Bunyakovsky$ ta có$(b+c)^2 \leq 2(b^2+c^2) \rightarrow b+c\leq \sqrt{2(b^2+c^2)}\rightarrow \frac{a^2}{b+c}\geq \frac{a^2}{\sqrt{2(b^2+c^2)}}$

CMTT $\rightarrow \frac{b^2}{a+c}\geq \frac{b^2}{\sqrt{2(a^2+c^2)}};\frac{c^{2}}{c+a}\geq \frac{c^2}{\sqrt{2(c^2+a^2)}}$

Đặt $x=\sqrt{b^2+c^2};y=\sqrt{c^2+a^2};z=\sqrt{a^2+b^2}\rightarrow x+y+z=\sqrt{2015}$

$\rightarrow P\geq \frac{y^2+z^2-x^2}{2\sqrt{2}}+\frac{z^2+x^2-y^2}{2\sqrt{2}}+\frac{x^2+y^2-z^2}{2\sqrt{2}}=\frac{1}{2\sqrt2}[(\frac{y^2+z^2}{x}-x)+(\frac{z^2+x^2}{y}-y)+(\frac{x^2+y^2}{z}-z)]$

Áp dụng BĐT $Bunyakovsky$ ta có

$2(y^2+z^2) \geq (y+z)^2 \rightarrow y^2+z^2 \geq \frac{(y+z)^2}{2}$

TT $\rightarrow z^2+x^2\geq \frac{(z+x)^2}{2};x^2+y^2\geq \frac{(x+y)^2}{2}$

$\rightarrow P\geq \frac{1}{2\sqrt2}[(\frac{(y+z)^2}{2x}+2x-3x)+(\frac{(z+x)^2}{2y}+2y-3y)+(\frac{(x+y)^2}{2z}+2z-3z)]$

Áp dụng BĐT $AM-GM$ ta có 

$\frac{(y+z)^2}{2x}+2x \geq 2(y+z); \frac{(z+x)^2}{2y}+2y \geq 2(x+z); \frac{(x+y)^2}{2z}+2z \geq 2(x+y)$

$\rightarrow P \geq \frac{1}{2\sqrt2}[2(y+z)-3x+2(z+x)-3y+2(x+y)-3z]=\frac{1}{2\sqrt2}(x+y+z)=\frac{1}{2}\sqrt{\frac{2015}{2}}$

 

P/s chậm hơn bạn NHoang1608 rồi @@


Bài viết đã được chỉnh sửa nội dung bởi tienduc: 31-03-2017 - 19:00


#108
Ren

Ren

    Hạ sĩ

  • Thành viên
  • 67 Bài viết

$\boxed{17}$ Giải phương trình $\sqrt{5-3x}+\sqrt{x+1}=\sqrt{3x^{2}-4x+4}$.

\[dkxd: - 1 \le x \le \frac{5}{3}\]

\[\sqrt {5 - 3x}  + \sqrt {x + 1}  = \sqrt {3{x^2} - 4x + 4} \]

\[ <  =  >  - 3{x^2} + 2x + 5 + 2\sqrt { - 3{x^2} + 2x + 5}  + 1 - 4 = 0\]

\[ <  =  > {\left( {\sqrt { - 3{x^2} + 2x + 5}  + 1} \right)^2} = 4\]

\[ <  =  > \sqrt { - 3{x^2} + 2x + 5}  + 1 = 2::or::\sqrt { - 3{x^2} + 2x + 5}  + 1 =  - 2(out)\]

\[ =  > \sqrt { - 3{x^2} + 2x + 5}  + 1 = 2 <  =  >  - 3{x^2} + 2x + 4 = 0\]

\[ <  =  > x = \frac{{1 + \sqrt {13} }}{3}(TM)::or::x = \frac{{1 - \sqrt {13} }}{3}(TM)\]



#109
NHoang1608

NHoang1608

    Sĩ quan

  • Thành viên
  • 375 Bài viết

Tiếp lửa cho $\boxed{TOPIC}$ nào  :ukliam2: :ukliam2:  :ukliam2:

$\boxed{19}$ Giải phương trình sau: $16x^{4}+5=6.\sqrt[3]{4x^{3}+x}$.

$\boxed{20}$ Tìm giá trị nhỏ nhất của biểu thức

                     $S=(3+\frac{1}{a}+\frac{1}{b})(3+\frac{1}{b}+\frac{1}{c})(3+\frac{1}{c}+\frac{1}{a})$

Trong đó $a, b, c$ là các số thực dương thỏa mãn điều kiện $a+b+c\leq \frac{3}{2}$.


Bài viết đã được chỉnh sửa nội dung bởi NHoang1608: 02-04-2017 - 09:42

The greatest danger for most of us is not that our aim is too high and we miss it, but that it is too low and we reach it.

----- Michelangelo----


#110
viet9a14124869

viet9a14124869

    Trung úy

  • Thành viên
  • 903 Bài viết

Tiếp lửa cho $\boxed{TOPIC}$ nào  :ukliam2: :ukliam2:  :ukliam2:

$\boxed{19}$ Giải phương trình sau: $16x^{4}+5=6.\sqrt[3]{4x^{3}+x}$.

$\boxed{20}$ Tìm giá trị nhỏ nhất của biểu thức

                     $S=(3+\frac{1}{a}+\frac{1}{b})(3+\frac{1}{b}+\frac{1}{c})(3+\frac{1}{c}+\frac{1}{a})$

Trong đó $a, b, c$ là các số thực dương thỏa mãn điều kiện $a+b+c\leq \frac{3}{2}$.

Bài 19 ,,, ^-^

ĐKXĐ x >0

Ta có $16x^4+5=16x^4+1+4\geq 8x^2+4=4x^2+(4x^2+1)+3\geq 4x^2+4x+3=(4x^2+1)+4x+2\geq 3\sqrt[3]{8x(4x^2+1)}=6\sqrt[3]{4x^3+x}\Rightarrow x=\frac{1}{2}$             


                                                                    SÓNG BẮT ĐẦU TỪ GIÓ

                                                                    GIÓ BẮT ĐẦU TỪ ĐÂU ?

                                                                    ANH CŨNG KHÔNG BIẾT NỮA 

                                                                    KHI NÀO...? TA YÊU NHAU .


#111
Minhnksc

Minhnksc

    Sĩ quan

  • Điều hành viên OLYMPIC
  • 302 Bài viết

Tiếp lửa cho $\boxed{TOPIC}$ nào  :ukliam2: :ukliam2:  :ukliam2:

$\boxed{19}$ Giải phương trình sau: $16x^{4}+5=6.\sqrt[3]{4x^{3}+x}$.

$\boxed{20}$ Tìm giá trị nhỏ nhất của biểu thức

                     $S=(3+\frac{1}{a}+\frac{1}{b})(3+\frac{1}{b}+\frac{1}{c})(3+\frac{1}{c}+\frac{1}{a})$

Trong đó $a, b, c$ là các số thực dương thỏa mãn điều kiện $a+b+c\leq \frac{3}{2}$.

$\boxed{20}$

theo bất đẳng thức AM-GM, ta có

$\frac{3}{2}\geq a+b+c\geq 3\sqrt[3]{abc} \Rightarrow abc\leq \frac{1}{8} \Rightarrow \frac{1}{abc}\geq 8$(1)

tiếp tục áp dụng bất đẳng thức AM-GM được

$\frac{1}{a}+\frac{1}{b}+3= \frac{1}{a}+\frac{1}{b}+\underbrace{\frac{1}{2}+\frac{1}{2}+...+\frac{1}{2}}_{6}\geq8\sqrt[8]{\frac{1}{2^6ab}}$

tương tự$ \frac{1}{c}+\frac{1}{b}+3\geq8\sqrt[8]{\frac{1}{2^6bc}}$

              $ \frac{1}{a}+\frac{1}{c}+3\geq8\sqrt[8]{\frac{1}{2^6ac}}$

từ đó ta có $S\geq 8^3\sqrt[8]{2^{18}\frac{1}{a^2b^2c^2}}$

kết hợp với (1)  suy ra $minS=64$

dấu bằng xảy ra $\Leftrightarrow a=b=c=\frac{1}{2}$


Bài viết đã được chỉnh sửa nội dung bởi Minhnksc: 02-04-2017 - 23:44

Sống khỏe và sống tốt :D


#112
Minhnksc

Minhnksc

    Sĩ quan

  • Điều hành viên OLYMPIC
  • 302 Bài viết

Góp 1 câu sử dụng Cauchy-schwarz 

$\boxed{21}$ Cho a,b,c là 3 cạnh của 1 tam giác, chứng minh rằng

$\frac{a}{b+c}+\frac{b}{a+c}+\frac{c}{a+b}+\frac{ab+bc+ca}{a^2+b^2+c^2}\leq \frac{5}{2}$


Bài viết đã được chỉnh sửa nội dung bởi Minhnksc: 03-04-2017 - 11:28

Sống khỏe và sống tốt :D


#113
viet9a14124869

viet9a14124869

    Trung úy

  • Thành viên
  • 903 Bài viết

Góp 1 câu sử dụng Cauchy-schwarz 

$\boxed{21}$ Cho a,b,c là 3 cạnh của 1 tam giác, chứng minh rằng

$\frac{a}{b+c}+\frac{b}{a+c}+\frac{c}{a+b}+\frac{ab+bc+ca}{a^2+b^2+c^2}\geq \frac{5}{2}$

Bài toán sai tại a=3,b=4,c=5 nhé bạn ,,,có lẽ nhìn qua ta sẽ nghĩ đề đúng ,,bởi có dấu = quen thuộc là a=b=c nhưng nếu a,b,c là 3 cạnh một tam giác không đều thì sẽ sai đấy bạn !!! 


                                                                    SÓNG BẮT ĐẦU TỪ GIÓ

                                                                    GIÓ BẮT ĐẦU TỪ ĐÂU ?

                                                                    ANH CŨNG KHÔNG BIẾT NỮA 

                                                                    KHI NÀO...? TA YÊU NHAU .


#114
Minhnksc

Minhnksc

    Sĩ quan

  • Điều hành viên OLYMPIC
  • 302 Bài viết

Bài toán sai tại a=3,b=4,c=5 nhé bạn ,,,có lẽ nhìn qua ta sẽ nghĩ đề đúng ,,bởi có dấu = quen thuộc là a=b=c nhưng nếu a,b,c là 3 cạnh một tam giác không đều thì sẽ sai đấy bạn !!.

mình sửa đề rồi đó bạn, lộn dấu =)).


Bài viết đã được chỉnh sửa nội dung bởi Minhnksc: 03-04-2017 - 11:29

Sống khỏe và sống tốt :D


#115
NHoang1608

NHoang1608

    Sĩ quan

  • Thành viên
  • 375 Bài viết

$\boxed{22}$ Giải phương trình sau: $(x+3)\sqrt{(4-x)(12+x)}=28-x$

$\boxed{23}$ Chứng minh rằng $\frac{a}{b}+\frac{b}{c}+\frac{c}{a} \geq \frac{3}{2}(a+b+c-1)$, trong đó $a,b,c$ là các số thực dương thỏa mãn điều kiện $abc=1$. Đẳng thức xảy ra khi nào?

 

 

P/s: Bài 21 hình như xử lí bằng S.O.S. :wacko:


Bài viết đã được chỉnh sửa nội dung bởi NHoang1608: 03-04-2017 - 17:24

The greatest danger for most of us is not that our aim is too high and we miss it, but that it is too low and we reach it.

----- Michelangelo----


#116
Nghiapnh1002

Nghiapnh1002

    Trung sĩ

  • Thành viên
  • 108 Bài viết

Bài 21: $(1-\frac{a}{b+c})+(1-\frac{b}{c+a})+(1-\frac{c}{a+b}) \geq \frac{1}{2}+\frac{ab+bc+ca}{2(a^2+b^2+c^2)}\Leftrightarrow \sum \frac{b+c-a}{b+c} \geq \frac{(a+b+c)^2}{2(a^2+b^2+c^2)}$ Do $a,b,c$ là các cạnh tam giác nên ta đảm bảo tử dương. Nên ta có:

$\sum \frac{b+c-a}{b+c} \geq \frac{[(b+c-a)+(c+a-b)+(a+b-a)]^2}{\sum(b+c)(b+c-a)}=\frac{(a+b+c)^2}{2(a^2+b^2+c^2)}\Rightarrow Q.E.D$

Bài 24:

$\sum \frac{1}{a(1+b)} \geq \frac{3}{\sqrt[3]{abc}(1+\sqrt[3]{abc})}$


Bài viết đã được chỉnh sửa nội dung bởi Nghiapnh1002: 05-04-2017 - 07:42


#117
Minhnksc

Minhnksc

    Sĩ quan

  • Điều hành viên OLYMPIC
  • 302 Bài viết

$\boxed{22}$ Giải phương trình sau: $(x+3)\sqrt{(4-x)(12+x)}=28-x$

$\boxed{23}$ Chứng minh rằng $\frac{a}{b}+\frac{b}{c}+\frac{c}{a} \geq \frac{3}{2}(a+b+c-1)$, trong đó $a,b,c$ là các số thực dương thỏa mãn điều kiện $abc=1$. Đẳng thức xảy ra khi nào?

 

 

P/s: Bài 21 hình như xử lí bằng S.O.S. :wacko:

$\boxed{22}$

ĐKXĐ:$-12\leq x\leq 4$

đặt $u=x+3$ và $v=\sqrt{(4-x)(12+x)}$

do đó phương trình ban đầu tương đương

$uv=\frac{u^2+v^2}{2}-\frac{1}{2}\Leftrightarrow (u-v)^2=1\Leftrightarrow $\begin{bmatrix} & x+3=\sqrt{(4-x)(x+12)}+1\\ & x+3=\sqrt{(4-x)(x+12)}-1 \end{bmatrix}$$

đến đây biến đổi và bình phương 2 vế là ra.


Bài viết đã được chỉnh sửa nội dung bởi Minhnksc: 09-04-2017 - 21:14

Sống khỏe và sống tốt :D


#118
Minhnksc

Minhnksc

    Sĩ quan

  • Điều hành viên OLYMPIC
  • 302 Bài viết

$\boxed{25}$

Với a,b,c là 3 số nguyên dương, chứng minh $(1+a^3)(1+b^3)(1+c^3)\geq (1+a^2b)(1+b^2c)(1+c^2a)$

P/s:một câu về BĐT Holder.


Bài viết đã được chỉnh sửa nội dung bởi Minhnksc: 04-04-2017 - 21:25

Sống khỏe và sống tốt :D


#119
viet9a14124869

viet9a14124869

    Trung úy

  • Thành viên
  • 903 Bài viết

$\boxed{25}$

Với a,b,c là 3 số nguyên dương, chứng minh $(1+a^3)(1+b^3)(1+c^3)\geq (1+a^2b)(1+b^2c)(1+c^2a)$

P/s:một câu về BĐT Holder.

$(1+a^3)(1+a^3)(1+b^3)\geq (1+a^2b)^3\Rightarrow (\prod (1+a^3))^3\geq (\prod (1+a^2b))^3\Rightarrow Q.E.D$


                                                                    SÓNG BẮT ĐẦU TỪ GIÓ

                                                                    GIÓ BẮT ĐẦU TỪ ĐÂU ?

                                                                    ANH CŨNG KHÔNG BIẾT NỮA 

                                                                    KHI NÀO...? TA YÊU NHAU .


#120
Minhnksc

Minhnksc

    Sĩ quan

  • Điều hành viên OLYMPIC
  • 302 Bài viết

$\boxed{26}$ Giải phương trình $x.\sqrt{x^2+x+1}+\sqrt{3x+1}=x^2+x+3$

$\boxed{27}$ Giải phương trình $x^3+5x^2+10x=(3x^2+3x+6)\sqrt{x+2}$

P/s: lâu rồi không qua topic, thấy vắng quá =((


Bài viết đã được chỉnh sửa nội dung bởi Minhnksc: 09-04-2017 - 18:10

Sống khỏe và sống tốt :D





1 người đang xem chủ đề

0 thành viên, 1 khách, 0 thành viên ẩn danh